Đăng ký Đăng nhập
Trang chủ Giáo dục - Đào tạo Toán học Bồi dưỡng học sinh giỏi môn toán thpt chuyên đề sử dụng số phức giải các toán về...

Tài liệu Bồi dưỡng học sinh giỏi môn toán thpt chuyên đề sử dụng số phức giải các toán về đa thức

.PDF
25
1257
77

Mô tả:

CHUYÊN ĐỀ SỬ DỤNG SỐ PHỨC GIẢI CÁC TOÁN VỀ ĐA THỨC Phạm Xuân Thịnh, trường THPT chuyên Hạ Long, Quảng Ninh 1 I. MỞ ĐẦU 0.1 Lí do chọn đề tài Đa thức là một đối tượng được nghiên cứu nhiều nhất trong Toán học. Ngay từ THCS, THPT đa thức đã được đề cập rất nhiều trong chương trình sách giáo khoa và sách chuyên khảo. Ở những bậc học này, đa thức thường được nghiên cứu về các tính chất Đại số, tính chất Số học và tính chất Giải tích. Đến bậc Đại học và sau Đại học, thì đa thức là một đối tượng cơ bản được nghiên cứu trong nhiều ngành Toán học khác nhau như Đại số, Lý thuyết số, Hình học, Giải tích, Toán rời rạc,... Có lẽ do tính quan trọng của đa thức trong Toán học mà các bài toán về đa thức thường xuất hiện nhiều trong các kì thi học sinh giỏi, nhất là thi học sinh giỏi Quốc gia và Quốc tế với mức độ thường được đánh giá là khó. Cái khó của các bài toán về đa thức nhiều khi là ta không biết bắt đầu từ đâu. Một vấn đề quan trọng đối với đa thức là vấn đề nghiệm của nó. Nhiều bài toán về đa thức được giải quyết dựa vào việc xử lí nghiệm của nó. Tuy nhiên trên trường số thực, một đa thức có thể có nghiệm nhưng cũng có thể vô nghiệm. Hơn nữa, thật khó để có thể biết một đa thức có nghiệm thực hay không. Vì thế việc giải quyết các bài toán đa thức thông qua nghiệm thực của nó sẽ trở nên vô cùng khó khăn. Nhưng trên trường số phức lại khác, vấn đề tồn tại nghiệm của đa thức không còn là sự trở ngại nữa. Bởi vì mọi đa thức khác hằng đều có nghiệm phức và do đó nếu ta xử lí bài toán thông qua các nghiệm phức của nó sẽ rất thuận lợi. Vì những lí do trên, chúng tôi quyết định chọn đề tài:”SỬ DỤNG SỐ PHỨC ĐỂ 1 Các trao đổi với tác giả về bài viết này có thể liên hệ qua email : [email protected] 2 GIẢI CÁC BÀI TOÁN VỀ ĐA THỨC” nhằm nghiên cứu sâu hơn về đa thức cũng như củng cố thêm kiến thức về số phức, phục vụ cho việc dạy và học. Với đề tài này, chúng tôi hi vọng phần nào chia sẻ và giúp các bạn có thêm cách tiếp cận các bài toán về đa thức. Ngoài ra, thông qua đề tài này, chúng tôi rất mong muốn nhận được những ý kiến đánh giá, những góp ý trao đổi của các đồng nghiệp, các em học sinh nói riêng và tất cả những ai quan tâm nói chung. 0.2 Mục đích và nhiệm vụ nghiên cứu Dựa trên thực tế giảng dạy và học tập, mục đích và nhiệm vụ nghiên cứu của đề tài là: i) Nghiên cứu những tính chất sâu hơn về đa thức. ii) Đề xuất một số biện pháp rèn luyện tư duy, năng lực giải toán và kĩ năng vận dụng số phức để giải các bài toán về đa thức như: chia đa thức, bài toán xác định đa thức, bài toán về đa thức bất khả quy, ... iii) Tạo tiền đề cho những nghiên cứu về sử dụng số phức và đa thức vào các bài toán tổ hợp. 0.3 Đối tượng học sinh Chuyên đề dùng để dạy đối tượng học sinh chuyên toán của trường THPT, đặc biệt là bồi dưỡng học sinh trong đội tuyển thi học sinh giỏi Quốc gia và Quốc tế. Ký hiệu và quy ước N : tập các số tự nhiên Z : tập các số nguyên Q : tập các số hữu tỉ R : tập các số thực C : tập các số phức Q+ (Q− ) : tập các số hữu tỉ không âm (không dương) Q∗ : tập các số hữu tỉ khác không vp (a) : số mũ cao nhất của số nguyên tố p trong biểu diễn chính tắc của số nguyên a. 3 II.NỘI DUNG 1 Kiến thức chuẩn bị Trong mục này chúng tôi nêu một số kiến thức về đa thức (không chứng minh) nhằm phục vụ cho các nội dung của chuyên đề. Giả sử K là một tập con khác rỗng của R. Kí hiệu K[x] là tập các đa thức với hệ số thuộc K. Định lý 1. Mọi đa thức bậc n (n ≥ 0) với hệ số thực đều có tối đa n nghiệm thực (tính cả bội). Định lý 2. Mọi đa thức bậc n (n ≥ 1) đều có đúng n nghiệm phức (tính cả bội). Định lý 3 (Định lí Bezout). Số thực a là nghiệm của đa thức P (x) ∈ R[x] khi và chỉ khi P (x) chia hết cho x − a, hay nói khác đi, tồn tại đa thức Q(x) ∈ R[x] sao cho P (x) = (x − a)Q(x), ∀x ∈ R. Định nghĩa 1. Đa thức P (x) ∈ K[x] được gọi là bất khả quy trên K nếu nó không thể phân tích thành tích của 2 đa thức khác hằng thuộc K[x]. Định lý 4. Cho P (x) ∈ Z[x]. Khi đó, P (x) bất khả quy trên Z khi và chỉ khi P (x) bất khả quy trên Q. Định lý 5. Nếu các biểu thức đối xứng cơ bản của các biến x1 , x2 , . . . , xn đều nhận giá trị nguyên (hữu tỉ), thì mọi đa thức đối xứng với hệ số nguyên của các biến x1 , x2 , . . . , xn cũng nhận giá trị nguyên (tương ứng: hữu tỉ). 2 2.1 Sử dụng số phức giải các bài toán về đa thức Bài toán xác định đa thức, chia hết Ví dụ 1. Tìm tất cả các đa thức P (x) ∈ R[x] thỏa mãn điều kiện P (x).P (2x2 ) = P (2x3 + x), ∀x ∈ R. 4 Giải. Nếu P (x) ≡ c là một đa thức hằng, thì từ giả thiết, ta có c2 = c hay c = 0 và c = 1. Xét P (x) là một đa thức khác hằng. Nếu P (x) có nghiệm thực α, thì 2α3 + α cũng là nghiệm của P (x). Xảy ra 2 trường hợp sau • Nếu α = 0, thì P (0) = 0. Đặt P (x) = xk .Q(x) với k ≥ 1, Q(0) 6= 0. Khi đó, 2k .x3k Q(x)Q(2x2 ) = (2x3 + x)k Q(2x3 + x) hay là 2k .x2k Q(x)Q(2x2 ) = (2x2 + 1)k Q(2x3 + x). Từ đây suy ra Q(0) = 0. Mâu thuẫn ! • Nếu α 6= 0, thì ta xét dãy số (un ) xác định bởi u1 = α, un+1 = 2u3n +un , n ≥ 1. Khi đó, P (un ) = 0, ∀n ≥ 1. Hơn nữa, dễ dàng thấy rằng dãy này tăng khi α > 0 và giảm khi α < 0. Điều đó chứng tở P (x) ≡ 0. Mâu thuẫn ! Vậy P (x) không có nghiệm thực. Do đó, deg(P (x)) = 2n, n ∈ N∗ . Vì P (0) = 1 nên tích tất cả các nghiệm của P (x) bằng 1, theo đó tích môđun của tất cả các nghiệm cũng bằng 1. Ta sẽ chứng minh tất cả các nghiệm của P (x) đều có môđun bằng 1. Thật vậy, nếu điều đó không đúng, gọi α là nghiệm phức có môđun lớn nhất trong các nghiệm của P (x), thì |α| > 1. Khi ấy, từ |2α2 + 1|2 = (2α2 + 1)(2α2 + 1) = 4|α|4 + 2(α2 + α2 ) + 1 = 4|α|2(|α|2 − 1) + 2(α + α)2 + 1 > 1, suy ra |2α3 +α| = |α|.|2α2 +1| > |α|. Mà 2α3 +α cũng là nghiệm của P (x). Điều này mâu thuẫn với cách chọn α. Bây giờ từ |α| = |2α3 + α| = 1, ta suy ra |2α2 + 1| = 1 hay (α + α)2 = 0, tức là α = −α. Do đó, α là số thuần ảo hay α = k.i, k ∈ R. Từ |α| = 1 ta có |k| = 1 hay k = ±1. Thành thử, α = ±i. Theo đó, P (x) chỉ có nghiệm là ±i và do P (x) là đa thức với hệ số thực nên bội của i và −i phải bằng nhau. Vì thế, P (x) = (x + i)n (x − i)n = (x2 + 1)n . Ví dụ 2. Tìm tất cả các đa thức P (x) ∈ R[x] khác hằng thỏa mãn P (x)P (x + 1) = P (x2 + 1), ∀x ∈ R. Giải. Trước hết ta thấy P (x) không có nghiệm thực. Thật vậy nếu P (x) có một nghiệm thực là a, thì a2 + 1, (a2 + 1)2 + 1, ... cũng là nghiệm. Vì a < a2 + 1 < (a2 + 1)2 + 1 < ... 5 nên P (x) có vô số nghiệm. Điều này là vô lí, vì P (x) là đa thức khác hằng. Do đó, deg(P (x)) = 2n, n ∈ N∗ . Gọi α là một nghiệm phức bất kì của P (x), thì dễ thấy n α2 + 1, α4 + 1, ..., α2 + 1, ... cũng là nghiệm của P (x). Vì P (x) là đa thức khác hằng phải tồn tại 2 số nguyên dương m, n, m < n sao cho  n m m n m α2 + 1 = α2 + 1 ⇐⇒ α2 α2 −2 − 1 = 0. Do α 6= 0 nên α2 −2 = 1 hay |α| = 1. Như vậy ta chứng minh được mọi nghiệm của P (x) đều có môđun bằng 1. Bây giờ nếu gọi α là một nghiệm của P (x), thì α2 + 1 và α2 − 2α + 2 cũng là nghiệm. Do đó, |α| = α2 + 1 = α2 − 2α + 2 = 1. n Chú ý là m 2 α + 1 2 = |α|4 + (α + α)2 − 2 |α|2 + 1 = (α + α)2 nên từ |α2 + 1| = 1 ta suy ra (α + α)2 = 1. Mặt khác, từ 2 α − 2α + 2 2 = |α|4 − (2 |α|2 + 4)(α + α) + 2(α + α)2 + 4 = 7 − 6(α + α) và |α2 − 2α + 2| = 1 ta có α + α = 1. kiện α.α = |α|2 = 1 ta suy ra √ Kết hợp với sự √ 3i 3i 1 1 và α = + và 2 nghiệm này phải có P (x) có đúng 2 nghiệm là α = + 2 2 2 2 bội bằng nhau. Vậy, P (x) = (x − α)n .(x − α)n = (x2 − x + 1)n . Ví dụ 3. Tìm tất cả các đa thức P (x) ∈ R[x] khác hằng sao cho P (x)P (x + 1) = P (x2 ), ∀x ∈ R. Giải. Gọi α là một nghiệm bất kì (thực hoặc phức) của P (x), thì dễ dàng thấy n rằng α2 , α4 , . . . , α2 , . . . cũng là nghiệm của P (x). Do P (x) không phải đa thức hằng nên tồn tại 2 số nguyên dương m, n(m < n) sao cho  n m m n m α2 = α2 ⇐⇒ α2 α2 −2 − 1 = 0. Do đó, hoặc α = 0 hoặc |α| = 1. 6 i) Nếu α = 0, thì từ phương trình trong đề bài, ta cho x = 0, được P (1) = 0, suy ra x = 1 cũng là nghiệm. ii) Nếu |α| = 1, thì từ giả thiết, bằng cách cho x = α − 1, ta thấy (α − 1)2 cũng là nghiệm, suy ra hoặc |(α − 1)2 | = 0 hoặc |(α − 1)2 | = 1. Nếu |(α − 1)2 | = 0, thì α = 1, còn nếu |(α − 1)2 | = 1, thì 1 = (α − 1)2 = |α − 1|2 = (α − 1)(α − 1) = 2 − (α + α), tức là α + α = 1. Bây giờ từ giả thiết ta cho x = α2 − 1, thì được nghiệm là (α2 − 1)2 , suy ra |α2 − 1| = 0 hoặc |α2 − 1| = 1. • Nếu |α2 − 1| = 0, thì α2 = 1 hay α = ±i. Trường hợp này α + α = 0. Mâu thuẫn với α + α = 1. • Nếu |α2 − 1| = 1, thì 2   1 = α2 − 1 = α2 − 1 α2 − 1 = 4 − (α + α)2 = 3. Điều này là vô lí ! Vậy trong mọi trường hợp, P (x) chỉ có nghiệm là 0 và 1. Do đó, P (x) = axm (x − 1)n , (a 6= 0; m, n ∈ N∗ ). Thay vào phương trình trong đề bài ta được a = 1 và m = n. Vậy P (x) = xn (x − 1)n . Ví dụ 4 (VMO, 2015). Cho (fn (x)) là dãy các đa thức xác định bởi f0 (x) = 2, f1 (x) = 3x, fn (x) = 3xfn−1 (x) + (1 − x − 2x2 )fn−2 (x), ∀n ≥ 2. Tìm tất cả các số nguyên dương n sao cho fn (x) chia hết cho x3 − x2 + x. √ Giải. Vì phương trình x3 − x2 + x = 0 có 3 nghiệm là x = 0 và x = 21 ± 23 i nên √ fn (x) chia hết cho x3 − x2 + x khi và chỉ khi fn (0) = 0 và fn 12 + 23 .i = 0. Từ giả thiết, ta suy ra fn (0) = fn−2 (0) và √ ! √ ! 1 3 3 3 3 fn + .i = + .i fn−1 2 2 2 2 √ ! √ ! 3 1 3 3 3 + .i + − .i fn−2 2 2 2 2 √ ! 3 1 + .i . 2 2 7 Từ f0 (0) = 2 o và f1 (0) = 0 ta suy ra fn (0) = 0 khi và chỉ khi n là số lẻ. Rõ ràng, n √ 1 3 là một dãy truy hồi tuyến tính cấp 2 có phương trình đặc trưng là fn 2 + 2 .i √ ! √ 3 3 3 3 3 3 + .i t − + .i = 0. t2 − 2 2 2 2 √ √ 3 3 Phương trình này có 2 nghiệm là t1 = + .i và t2 = 3.i. Từ đây dễ dàng tính 2 2 được " # !n ! √ √ √ n 1 3 3 i fn + in . + .i = ( 3) + 2 2 2 2 √   √ 3 i π π π π Viết + = cos + i. sin và i = cos + i. sin . Khi đó, fn 21 + 23 .i = 0 2 2 6 6 2 2 nếu và chỉ nếu  cos nπ + cos nπ = 0   nπ nπ nπ nπ 6 2 cos = 0 ⇐⇒ + cos + i sin + sin sin nπ + sin nπ = 0 6 2 6 2 6 2 ⇐⇒ n = 3(2k + 1). Vậy tất cả các số nguyên dương của n để fn (x) chia hết cho x3 − x2 + x là n = 3(2k + 1), k ∈ N. Ví dụ 5. Cho đa thức f (x) ∈ R[x] sao cho f (x) ≥ 0, ∀x ∈ R. Chứng minh rằng tồn tại các đa thức g(x), h(x) ∈ R[x] sao cho f (x) = g 2 (x) + h2 (x), ∀x ∈ R. Giải. Gọi hệ số cao nhất của f (x) là a. Vì f (x) ≥ 0, ∀x ∈ R nên a > 0. Hơn nữa, nếu f (x) có nghiệm thực x = a, thì nghiệm này phải có số bội chẵn. Do đó, ta có thể giả sử f (x) = a(x − x1 )2k1 (x − x2 )2k2 . . . (x − xm )2km g(x), trong đó x1 , x2 , . . . , xm là tất cả các nghiệm thực (nếu có) của f (x) và g(x) là đa thức monic không có nghiệm thực. Ngoài ra, do f (x) ≥ 0, ∀x ∈ R nên g(x) > 0, ∀x ∈ R. Thế thì g(x) phải có bậc chẵn. Đặt deg(g(x)) = 2n, thì g(x) có 2n nghiệm phức. Chú ý là do g(x) ∈ R[x] nên nếu α là một nghiệm phức của g(x), thì α cũng là nghiệm của g(x) với số bội bằng số bội của α. Gọi α1 , α2 , . . . , αn , α1 , α2 , . . . , αn là tất cả các nghiệm phức của g(x), thì g(x) phân tích được thành g(x) = [(x − α1 )(x − α1 )]h1 .[(x − α2 )(x − α2 )]h2 . . . [(x − αn )(x − αn )]hn . 8 Với mỗi j = 1, 2, . . . , n ta viết αj = aj + bj .i, aj , bj ∈ R thì (x − αj )(x − αj ) = x2 − (αj + αj )x + αj .αj x2 − 2aj x + a2j + b2j = (x − aj )2 + b2j . Vì tích của các tổng 2 bình phương lại là tổng 2 bình phương nên g(x) có thể viết thành g(x) = u2 (x) + v 2 (x), ∀x ∈ R, ở đó u(x), v(x) ∈ R[x]. Bây giờ nếu đặt m m √ Y √ Y kj g(x) = a. (x − xj ) .u(x) và h(x) = a. (x − xj )kj .v(x), j=1 j=1 thì f (x) = g 2 (x) + h2 (x), ∀x ∈ R. Dưới đây là một số bài tập có thể sử dụng số phức để giải. Bài 1 (Biên của nghiệm). Cho đa thức P (x) = an xn + an−1 xn−1 + · · · + ax + a0 ∈ Z[x], an 6= 0. Đặt M = max {|ai | : i = 0, 1, 2, . . . , n − 1} . Chứng minh rằng nếu α M là một nghiệm của P (x), thì |α| < 1 + . |an | Bài 2. Tìm tất cả các số nguyên dương n sao cho đa thức f (x) = (x + 1)n + xn + 1 chia hết cho g(x) = x2 + x + 1. Bài 3. Tìm tất cả các đa thức P (x) ∈ R[x] sao cho P (x)P (x − 3) = P (x2 ), ∀x ∈ R. Bài 4. Tìm tất cả các đa thức P (x) ∈ R[x] sao cho P (x)P (x + 1) = P (x2 + x + 1), ∀x ∈ R. Bài 5. Cho các đa thức P (x), Q(x), R(x), S(x) ∈ R[x] thỏa mãn P (x5 ) + x.Q(x5 ) + x2 .R(x5 ) = (1 + x + x2 + x3 + x4 )S(x), ∀x ∈ R. Chứng minh rằng P(x) chia hết cho x − 1. 9 2.2 Chứng minh đa thức bất khả quy Để chứng minh một đa thức P (x) ∈ Z[x] là bất khả quy trên Z ta hãy sử dụng phương pháp phản chứng, tức là giả sử P (x) là khả quy. Khi ấy, tồn tại các đa thức f (x), g(x) ∈ Z[x] có bậc lớn hơn hoặc bằng 1 sao cho P (x) = f (x).g(x), ∀x ∈ R. Từ đây bằng cách sử dụng các kết quả về đa thức, đặc biệt kết quả về nghiệm của đa thức, ta tìm được một điều vô lí hay mâu thuẫn nào đó và kết thúc chứng minh. Sau đây là các ví dụ minh họa. Trong suốt mục này nếu không nói gì thêm, ta hiểu bất khả quy (khả quy) nghĩa là bất khả quy (khả quy) trên Z. Ví dụ 1 (JMO,1999). Cho n là số nguyên dương. Chứng minh rằng đa thức P (x) = (x2 + 12 )(x2 + 22 ) . . . (x2 + n2 ) + 1 là bất khả quy. Giải. Giả sử f (x) khả quy trên Z, tức là f (x) = g(x).h(x), với g(x), h(x) ∈ Z[x] và có bậc lớn hơn hoặc bằng 1. Thế thì g(k.i).h(k.i) = f (k.i) = 1, ∀k ∈ Z, 1 ≤ |k| ≤ n. Vì g(x), h(x) ∈ Z[x] nên g(k.i) và h(k.i) có dạng a + b.i với a, b ∈ Z, suy ra |g(k.i)|2 và |h(k.i)|2 là các số tự nhiên. Do đó, từ |g(k.i)|2 .|h(k.i)|2 = 1 suy ra |g(k.i)| = 1 và h(k.i) = 1 = g(k.i) = g(k.i) = g(−k.i). g(k.i) Điều này dẫn tới đa thức h(x) − g(−x) có 2n nghiệm. Hơn nữa, đa thức này có bậc nhỏ hơn 2n nên h(x) = g(−x), ∀x. Như thế, f (x) = g(x).g(−x) và do đó, f (0) = [g(0)]2 , tức là (n!)2 + 1 = [g(0)]2 . Dễ thấy rằng điều này là không thể. Vậy điều giả sử là sai hay f (x) là đa thức bất khả quy. Ví dụ 2 (IMO,1993). Chứng minh rằng đa thức f (x) = xn + 5xn−1 + 3 bất khả quy trên Z. Giải. Giả sử f (x) là một đa thức khả quy, tức là tồn tại g(x), h(x) ∈ Z[x] có bậc lớn hơn hoặc bằng 1 sao cho f (x) = g(x).h(x). Vì g(−5).h(−5) = f (−5) = 3 nên hoặc |g(−5)| = 1 hoặc |h(−5)| = 1. Giả sử |g(−5)| = 1. Đặt k = deg(g(x)) và gọi α1 , α2 , . . . , αk là các nghiệm phức của g(x), thì g(x) = (x − α1 )(x − α2 ) . . . (x − αk ). 10 Từ đây suy ra |(α1 + 5)(α2 + 5) . . . (αk + 5)| = |g(−5)| = 1. Vì f (αi ) = 0 nên αin−1 (αi + 5) = −3, ∀i = 1, k. Do đó, |α1 .α2 . . . αk |n−1 .|(α1 + 5)(α2 + 5) . . . (αk + 5)| = 3k . Để ý là |α1 .α2 . . . αk | = |g(0)| và g(0).h(0) = f (0) = 3 nên |g(0)| = 1 hoặc |g(0)| = 3. Vì 3k > 1 nên |g(0)| > 1, suy ra |g(0)| = 3. Khi ấy, 3n−1 = 3k hay k = n − 1 =⇒ deg(h(x)) = 1, tức là f (x) phải có nghiệm nguyên x = a. Điều đó là không thể xảy ra. Vậy f (x) là đa thức bất khả quy. Ví dụ 3. Cho a, m, n là các số nguyên dương và số nguyên tố p < a − 1. Chứng minh rằng đa thức P (x) = xm (x − a)n + p bất khả quy. Giải. Giả sử P (x) khả quy. Thế thì tồn tại các đa thức f (x), g(x) ∈ Z[x] có bậc lớn hơn hoặc bằng 1 sao cho P (x) = f (x)g(x), ∀x ∈ R. Vì p = P (0) = f (0)g(0) nên có thể giả sử f (0) = 1. Đặt k = deg(f (x)) và gọi x1 , x2 , . . . , xk là các nghiệm phức của f (x). Do P (x) là monic nên có thể giả sử f (x) cũng monic. Khi ấy, f (x) = (x − x1 )(x − x2 ) . . . (x − xk ). Từ đây suy ra |x1 x2 . . . xk | = |f (0)| = 1. Vì xi , (1 ≤ i ≤ k) cũng là nghiệm của n P (x) nên xm i (xi − a) = −p. Do đó, n m n m n k |f (a)|n = |(a−x1 )(a−x2 )...(a−xk )|n = |xm 1 (x1 −a) |.|x2 (x2 −a) |...|xk (xk −a) | = p . Từ đây suy ra |f (a)| = p và k = n. Theo đó, f (a) − f (0) ∈ {p − 1, p + 1, −p + 1, −p − 1} . Mà a | f (a) − f (0) nên a | p − 1 hoặc a | p + 1. Nhưng cả hai tình huống này đều không xảy ra, vì 0 < p − 1 < p + 1 < a. Vậy điều giả sử là sai hay P (x) phải là một đa thức bất khả quy. Ví dụ 4. Cho p là số nguyên tố và an an−1 ...a1 a0 , an > 1 là biểu diễn của p trong hệ thập phân. Chứng minh rằng đa thức P (x) = an xn + an−1 xn−1 + · · · + a1 x + a0 bất khả quy. Giải. Giả sử ngược lại P (x) là đa thức khả quy. Khi đó tồn tại Q(x), H(x) ∈ Z[x] có bậc lớn hơn hoặc bằng 1 sao cho P (x) = Q(x).H(x). Vì P (10) = an .10n + an−1 .10n−1 + · · · + a1 .10 + a0 = p 11 nên hoặc |Q(10)| = 1 hoặc |H(10)| = 1. Giả sử |Q(10)| = 1 và gọi x1 , x2 , ..., xk , với k = deg(Q(x)) là tất cả các nghiệm phức của Q(x), thì Q(x) = a(x − x1 )(x − x2 )...(x − xk ), a ∈ Z, a 6= 0. Đặt M = max {a0 , a1 , ..., an−1 } , thì dễ dàng chứng M 9 minh được |xi | ≤ 1 + ≤ 1 + < 6. Từ đó suy ra an 2 1 = |Q(10)| = |a|.|10 − x1 |.|10 − x2 |...|10 − xk | > 1. Điều vô lí này chứng tỏ P (x) là bất khả quy. Ví dụ 5. Cho n ∈ N∗ . Chứng minh rằng đa thức f (x) = (x2 − 7x + 6)2n + 13 bất khả quy. Giải. Giả sử f (x) là đa thức khả quy, tức là tồn tại g(x), h(x) ∈ Z[x] có bậc lớn hơn hoặc bằng 1 sao cho f (x) = g(x).h(x). Gọi xi , i = 1, k là tất cả các nghiệm phức của g(x), trong đó k = deg(g(x)). Khi ấy, g(x) = (x − x1 )(x − x2 ) . . . (x − xk ). Từ đây suy ra và Do đó, |g(1)| = |(x1 − 1)(x2 − 1) . . . (xk − 1)| , |g(6)| = |(x1 − 6)(x2 − 6) . . . (xk − 6)| . 2 (x − 7x1 + 6)(x2 − 7x2 − 6) . . . (x2 − 7xk + 6) 2n = g 2n (1).g 2n (6). 1 2 k Vì xi , i = 1, k cũng là nghiệm của g(x) nên (x2i − 7xi + 6)2n = −13. Thành thử, 2 (x − 7x1 + 6)(x2 − 7x2 − 6) . . . (x2 − 7xk + 6) 2n = 13k . 1 2 k Như thế, g 2n (1).g 2n (6) = 13k . Vì 13 là số nguyên tố nên k = 2n, suy ra |g(1)| . |g(6)| = 13. Nhưng 5 | g(6) − g(1) nên g(6) = g(1). Suy ra 13 = g 2 (1). Vô lí ! Chú ý. Ta có vài chú ý sau về bài toán này. i) Bài toán trên là một kết quả mạnh hơn bài toán trong kì thi VMO năm 2014: ”Cho đa thức P (x) = (x2 − 7x + 6)2n + 13, n ∈ N∗ . Chứng minh rằng P (x) không thể phân tích thành tích của n+ 1 đa thức khác hằng với hệ số nguyên.” 12 ii) Nếu thay 13 bởi một số nguyên tố p sao cho p 6≡ ±1 (mod 5), thì kết quả vẫn không thay đổi. Ví dụ 6. Cho n là các số nguyên dương và p là một số nguyên tố. Chứng minh rằng đa thức f (x) = xp − x + pn bất khả quy. Giải. Giả sử f (x) khả quy, tức là tồn tại các đa thức g(x), h(x) ∈ Z[x] có bậc lớn hơn hoặc bằng 1 sao cho f (x) = g(x).h(x), ∀x ∈ R. Từ pn = f (0) = g(0).h(0) suy ra |g(0)| = pa và |h(0)| = pb . Không mất tính tổng quát, ta có thể coi 0 ≤ a ≤ b. Viết g(x) = xk + ak−1 xk−1 + · · · + a1 x + a0 và h(x) = xm + bm−1 xm−1 + · · · + b1 x + b0 , trong đó a0 = ±pa và b0 = ±pb . Bằng cách đồng nhất hệ số, ta có −1 = a1 b0 + a0 b1 = pa (±b1 ± a1 pb−a ). Suy ra a = 0 và do đó, |g(0)| = 1. Gọi x1 , x2 , . . . , xk là các nghiệm của g(x), thì g(x) = (x − x1 )(x − x2 ) . . . (x − xk ) =⇒ |x1 x2 . . . xk | = |g(0)| = 1. Từ đây ta thấy phải tồn tại i (1 ≤ i ≤ k) sao cho |xi | ≤ 1. Do xi cũng là nghiệm của f (x) nên pn = |xi − xpi | ≤ |xi | + |xi |p ≤ 2. Điều này chỉ có thể xảy ra nếu p = 2 và n = 1. Khi đó, f (x) = x2 − x + 2. Nhưng đây không phải là một đa thức khả quy. Vậy điều giả sử là sai hay f (x) phải là một đa thức bất khả quy. Ví dụ 7. Cho p là số nguyên tố và a là số nguyên không chia hết cho p. Chứng minh rằng P (x) = xp − x + a bất khả quy. Giải. Giả sử P (x) khả quy. Khi ấy tồn tại Q(x), H(x) ∈ Z[x] có bậc lớn hơn hoặc bằng 1 sao cho P (x) = Q(x).H(x). Đặt k = deg(Q(x)) và gọi x1 , x2 , . . . , xk là tất cả các nghiệm phức của Q(x). Ta có xpi = xi − a, ∀i = 1, k. Do đó xp1 + xp2 + · · · + xpk = x1 + x2 + · · · + xk − k.a Vì Q(x) ∈ Z[x] nên theo định lí Vieta, thì các đa thức đối xứng cơ bản của x1 , x2 , . . . , xk đều là số nguyên. Do đó tất cả các đa thức đối xứng đối với các biến 13 x1 , x2 , . . . , xk đều là số nguyên. Nói riêng, x1 +x2 +· · ·+xk ∈ Z và xp1 +xp2 +· · ·+xpk ∈ Z. Theo định lí nhỏ Fermat, ta có (x1 + x2 + · · · + xk )p ≡ x1 + x2 + · · · + xk (mod p). Mặt khác, ta lại có xp1 + xp2 + · · · + xpk = (x1 + x2 + · · · + xk )p − X 0≤i1 ,i2 ,...,ik

|a1 | + |a2 | + · · · + |an |. Chứng minh rằng P (x) là bất khả quy trên Z. Giải. Giả sử P (x) khả quy. Khi ấy tồn tại Q(x), H(x) ∈ Z[x] có bậc lớn hơn hay bằng 1 sao cho P (x) = Q(x).H(x). Vì |P (0)| = |a0 | là số nguyên tố nên |Q(0)| = 1 hoặc |H(0)| = 1. Giả sử |Q(0)| = 1. Gọi x1 , x2 , ..., xk là tất cả các nghiệm phức của Q(x), k = deg(Q(x)), thì Q(x) = a.(x − x1 )(x − x2 ) . . . (x − xk ), với a | an . Vì 1 ≤ 1. Do đó, tồn tại i, 1 ≤ i ≤ k sao cho |xi | ≤ 1. |Q(0)| = 1 nên |x1 |.|x2 |...|xk | = |a| Rõ ràng, xi cũng là nghiệm của P (x) nên P (xi ) = 0 hay  a0 = −xi a1 + a2 xi + · · · + an xin−1 . Từ đó suy ra |a0 | = |xi | . a1 + a2 xi + · · · + an xin−1 ≤ |a1 | + |a2 | + · · · + |an | . Điều này mâu thuẫn với giả thiết. Ví dụ 9. Cho p là một số nguyên tố, k, h ∈ N∗ sao cho rằng f (x) = k.xp − h là đa thức bất khả quy. r p h ∈ / Q. Chứng minh k Giải. Ta chỉ cần xét (h, k) = 1. Giả sử f (x) = g(x).h(x), với g(x), h(x) ∈ Z[x] và có bậc lớn hơn hoặc bằng 1. Đặt m = deg(g(x)), 1 ≤ m < pvà gọi ε1 , ε2 , . . . , εm 14 là các nghiệm phức của g(x). Khi đó, εi , ∀i = 1, m là các căn bậc p của g(x) = a(x − ε1 )(x − ε2 ) . . . (x − εm ), với a là ước dương của k. Ta có h và k h = |f (0)| = |g(0)|.|h(0)| = ab|ε1 |.|ε2| . . . |εm |, trong đó b = |h(0)| là ước dương của h. Từ đó suy ra p p p h = a .b .|εp1 |.|εp2| . . . |εpm |  m h = a .b . k p p ( k = c.au thì aum .bv(p−m) .cm .dp−m = ap .bp . Vì Suy ra hp−m .k m = ap .bp . Đặt h = d.bv a, b, c, d đôi một nguyên tố cùng nhau nên   p m=1 a = aum     (   b = 1 bp = bv(p−m) h=1 =⇒ =⇒   c=1 k = ap cm = 1       p−m d =1 d=1 Thành thử, r p 1 h = ∈ Q. Mâu thuẫn ! k a Ví dụ 10. Cho p là số nguyên tố dạng 4k + 3. Chứng minh rằng đa thức f (x) = (x2 + 1)n + p là bất khả quy. Giải. Giả sử ngược lại f (x) = g(x)h(x), trong đó g(x), h(x) ∈ Z[x] và có bậc lớn hơn hoặc bằng 1. Ta có p = h(i) = g(i)h(i) =⇒ p2 = |g(i)|2.|h(i)|2 (1) Vì các số |g(i)|2 và |h(i)|2 đều có dạng a2 + b2 với a, b ∈ Z và p là số nguyên tố dạng 4k + 3 nên không thể xảy ra trường hợp |g(i)|2 = |h(i)|2 = p. Do đó, ta có thể giả sử |g(i)| = 1. Đặt k = deg(g(x)) và gọi x1 , x2 , ..., xk là nghiệm của g(x), thì từ f (x) là monic ta suy ra g(x) cũng vậy. Hơn nữa, ta có g(x) = (x − x1 )(x − x2 ) . . . (x − xk ). Khi đó, 1 = |g(i)|2 = g(i)g(i) = g(i)g(−i) = (x21 + 1)(x22 + 1) . . . (x2k + 1). 15 Để ý là xj , j = 1, 2, . . . , k cũng là nghiệm của f (x) nên x2j + 1 có 1 = (x21 + 1)n (x22 + 1)n . . . (x2k + 1)n = (−p)k . n = −p. Do đó, ta Đây là điều vô lí ! Ví dụ 11 (Việt Nam TST, 2013). Tìm tất cả các số nguyên n > 1 và số nguyên tố p sao cho đa thức f (x) = xn − px + p2 khả quy trên Z. Giải. Giả sử f (x) = g(x)h(x), trong đó g(x), h(x) ∈ Z[x] có bậc lần lượt là k, m ≥ 1 (k + m = n). Từ p2 = f (0) = g(0).h(0) suy ra |g(0)| ∈ {1; p; p2 } . Nhưng thực tế ta chỉ cần quan tâm đến các trường hợp |g(0)| = 1 hoặc |g(0)| = p. Gọi x1 , x2 , . . . , xk là các nghiệm phức của g(x), thì g(x) = (x − x1 )(x − x2 ) . . . (x − xk ). • Nếu |g(0)| = 1, thì |x1 x2 . . . xk | = 1. Khi đó, do xni = p(xi − p), ∀i = 1, k nên 1 = |x1 x2 . . . xk |n = pk . |(x1 − p)(x2 − p) . . . (xk − p)| = pk . |g(p)| . Đây là điều vô lí ! • Nếu g(0) = h(0) = ±p, thì từ |x1 x2 . . . xk | = |g(0)| = p ta suy ra pn = |x1 x2 . . . xk |n = pk . |(x1 − p)(x2 − p) . . . (xk − p)| = pk . |g(p)| . Thành thử, |g(p)| = pm . Khi đó, ta có |h(p)| = pk . Do |g(0)| = |h(0)| = p nên g(−p) và h(−p) đều chia hết cho p. Mà   g(−p).h(−p) = f (−p) = p2 2 + (−p)n−2 nên g(−p) = a.p và h(−p) = b.p, trong đó a, b ∈ Z sao cho ab = 2 + (−p)n−2 . Ta có ( ( h(p) + h(−p) = 2g(0) + B.p2 g(p) + g(−p) = 2g(0) + A.p2 và h(p) + h(−p) = b.p + pk g(p) + g(−p) = a.p + pm với A, B ∈ Z. Từ đây suy ra nếu m ≥ 2 và k ≥ 2, thì  ( ( a.p − 2g(0) ... p2 a ≡ −2 (mod p) a ≡ 2 (mod p) hoặc =⇒ b.p − 2g(0) ... p2 b ≡ −2 (mod p) b ≡ 2 (mod p) 16 Cả hai trường hợp ta đều có ab ≡ 4 (mod p), suy ra 2 + (−p)n−2 ≡ 4 (mod p) . . hay p = 2. D đó, a và b là các số chẵn nên ab .. 4 hay 2 + (−2)n−2 .. 4. Nhưng điều này không xảy ra vì n = m + k ≥ 4. Như vậy bắt buộc phải có k = 1 hoặc m = 1, tức là f (x) phải có nghiệm nguyên x0 . Từ xn0 = p(x0 − p) suy ra . x0 .. p hay x0 = t.p. Do đó, tn pn−2 = t − 1. Phương trình này chỉ có nghiệm t = −1, p = 2, n = 3. Thử lại ta thấy n = 3, p = 2 thỏa mãn đề bài và đó là tất cả các giá trị cần tìm. Ví dụ 12. Cho p là số nguyên tố có dạng 4k + 3 và a, b là các số nguyên sao cho min {vp (a), vp (b − 1)} = 1. Chứng minh rằng đa thức f (x) = x2p + a.x + b là bất khả quy. Giải. Giả sử f (x) khả quy, tức là f (x) = g(x).h(x) với g(x), h(x) ∈ Z[x] và có bậc lớn hơn hoặc bằng 1. Ta có |f (i)|2 = |b − 1 + a.i|2 = a2 + (b − 1)2 . ( b − 1 = c.pu Đặt a = d.pv Khi ấy (với c, d ∈ Z; (c, p) = (d, p) = 1 và u, v ∈ N∗ ; min {u, v} = 1).  a2 + (b − 1)2 = p2 (c2 + d2 .p2v−2 ) a2 + (b − 1)2 = p2 (c2 .p2u−2 + d2 ) Vì (c, p) = (d, p) = 1 và p là số nguyên tố có dạng 4k+3 nên c2 +d2 .p2v−2 và c2 .p2u−2 +  d2 đều không chia hết cho p. Do đó, vp |f (i)|2 = 2. Rõ ràng là |g(i)|2 , |h(i)|2 ∈ N∗ . Do đó, từ |g(i)|2 . |h(i)|2 = |f (i)|2 và p là số nguyên tố có dạng 4k + 3 nên chỉ 2 2 có thể xảy ra hoặc vp |g(i)| = 2 hoặc vp |h(i)| = 2. Giả sử vp |g(i)|2 = 2 thì  vp |h(i)|2 = 0. Đặt k = deg(h(x)) và gọi x1 , x2 , ..., xk là tất cả các nghiệm phức của Q Q h(x) thì dễ thấy h(x) là monic và h(x) = kj=1 (x − xj ). Ta có h(i) = kj=1 (i − xj ), suy ra p k k Y Y 2p 2 2 2 (xj + 1) = |h(i)| =⇒ (xj + 1) = |h(i)| . j=1 j=1 P l 2l a 1 p−1 b−1 p Để ý là f (x) = (x2 + 1) − p.g(x) với g(x) = . Từ đó suy ra Cp x − x − p l=1 p p  p x2j + 1 = p.g(xj ), ∀j = 1, k. Như thế, ta có p k k Y Y 2p (x2j + 1) = |h(i)| = pk g(xj ) , (∗). j=1 j=1 17 Vì h(x) ∈ Z[x] nên theo định lí Vieta thì các đa thức đối xứng cơ bản của x1 , x2 , ..., xk là các số nguyên. Do đó, tất cả các đa thức đối xứng với hệ số nguyên của x1 , x2 , ..., xk Q cũng là số nguyên. Dễ thấy g(x) ∈ Z[x] nên kj=1 g(xj ) là số nguyên. Thành thử từ (∗) suy ra |h(i)|2p chia hết cho p. Điều này là vô lí ! Ví dụ 13. Chứng minh rằng đa thức f (x) = (x2 + x)2 + 1 bất khả quy với mọi n ∈ N∗ n Giải. Giả sử f (x) = g(x).h(x), trong đó g(x), h(x) ∈ Z[x] có bậc lớn hơn hoặc bằng 1. √ có thể giả thiết g(x) √ và h(x) cũng vậy. Đặt ε = √ Vì f (x) là monic nên a + 3b.i c + 3d.i −1 + 3i , thì g(ε) = và h(ε) = , trong đó a, b, c, d ∈ Z và 2 2k 2k (a, b, 2) = (c, d, 2) = 1. Từ   2 = f (ε) = g(ε).h(ε) =⇒ a2 + 3b2 c2 + 3d2 = 4k+m+1 . Do phương trình x2 + 3y 2 = 2 không có nghiệm nguyên nên ( a2 + 3b2 = 4u (u + v = k + m + 1). c2 + 3d2 = 4v Ta có thể giả sử u ≤ v. Khi ấy xảy ra 2 trường hợp sau • Nếu u = 0, thì a2 = 1, b = 0. Khi ấy v > 0 và do đó c, d phải có cùng lẻ, suy ra c2 + 3d2 ≡ 4 (mod 8) =⇒ v = 1. Từ đây ta có k + m = 0 ⇐⇒ k = m = 0 hay |g(ε)| = 1 và |h(ε)| = 2. • Nếu u > 0, thì v > 0. Lập luận tương tự như trên, ta được u = v = 1 hay k + m = 1. Do đó hoặc k = 1, m = 0 hoặc k = 0, m = 1. Thành thử hoặc |g(ε)| = 1 và |h(ε)| = 2 hoặc là |g(ε)| = 2 và |h(ε)| = 1. Tóm lại, không mất tính tổng quát, ta có thể giả sử |g(ε)| = 1. Đặt t = deg(g(x)) và gọi x1 , x2 , ..., xt là các nghiệm phức của g(x), thì g(x) = (x − x1 )(x − x2 ) . . . (x − xt ) =⇒ |g(ε)| = |(x1 − ε)(x2 − ε)...(xt − ε)| . Từ đây suy ra (x21 + x1 + 1)(x22 + x2 + 1) . . . (x2t + xt + 1) = |g(ε)|2 = 1. 18 Đặt q(x) = 2n P C2kn (−1)k (x2 + x + 1)k−1 , thì do k=1 n (x2i + xi + 1)q(xi ) + 2 = (x2i + xi )2 + 1 = 0, ∀i = 1, t nên ta có |g(ε)|2 .q(x1 )q(x2 ) . . . q(xt ) = (−2)t . Vì q(x) ∈ Z[x] nên các biểu thức đối xứng cơ bản của x1 , x2 , ..., xt là các số nguyên, suy ra các biểu thức đối xứng đối với x1 , x2 , ..., xt cũng vậy. Nói riêng, . q(x1 )q(x2 ) . . . q(xt ) là một số nguyên. Để ý là C kn .. 2, ∀k = 1, 2n nên ta đặt n −1 q(x) = (x2 + x + 1)2 + 2.k(x). Do đó, n −1) q(x1 )q(x2 ) . . . q(xt ) = |g(ε)|2(2 2 + 2.k(x1 , x2 , ..., xt ) = 1 + 2.k(x1 , x2 , ..., xt ), với k(x1 , x2 , ..., xt ) là biểu thức đối xứng đối với x1 , x2 , ..., xt . Thành thử, k(x1 , x2 , ..., xt ) ∈ Z và q(x1 )q(x2 ) . . . q(xt ) là một số lẻ. Do đó, (−2)t là số nguyên lẻ. Đây là điều vô lí, vì t ≥ 1. 2.3 Chứng minh một số tiêu chuẩn bất khả quy bằng số phức Trong mục này chúng tôi sẽ sử dụng số phức để chứng minh một số kết quả quan trọng về đa thức bất khả quy. Kết quả đầu tiên là tiêu chuẩn bất khả quy của Eisenstein. Đây là kết quả rất đẹp và quan trọng. Nhưng trong các tài liệu mà chúng tôi biết, thì chưa có một tài liệu nào chứng minh bằng số phức. Sau khi cố gắng để tìm một lời giải bằng số phức cho bài toán về đa thức bất khả quy trong kì thi chọn đội tuyển dự thi IMO năm 2013, chúng tôi đã mạnh dạn tìm một chứng minh bằng số phức cho tiêu chuẩn này và sau một thời gian suy nghĩ, chúng tôi đã tìm được một chứng minh đẹp đẽ và khá là đơn giản như trình bày dưới đây. Định lý 1 (Tiêu chuẩn Eisenstein). Cho P (x) = an xn + an−1 xn−1 + · · · + a1 x + a0 ∈ Z[x]. Nếu tồn tại một số nguyên tố p sao cho 3 điều kiện sau đồng thời thỏa mãn i) a0 , a1 , . . . , an−1 chia hết cho p; ii) an không chia hết cho p; iii) a0 không chia hết cho p2 , 19 thì P (x) là đa thức bất khả quy trên Z. Chứng minh. Giả sử P (x) không phải là đa thức bất khả quy. Khi ấy tồn tại các đa thức f (x), g(x) ∈ Z[x] có bậc lớn hơn hoặc bằng 1 sao cho P (x) = f (x)g(x), ∀x ∈ R. Vì a0 = P (0) = f (0)g(0) và vp (a0 ) = 1 nên ta có thể giả sử f (0) = b, với b | a0 , (b, p) = 1. Đặt k = deg(f (x)) và gọi x1 , x2 , . . . , xk là các nghiệm (thực hoặc phức) của f (x), thì f (x) = c(x − x1 )(x − x2 ) . . . (x − xk ), trong đó c là một số nguyên và c | an . Ta có b = f (0) = c.(−1)k x1 x2 . . . xk =⇒ x1 x2 . . . xk = (−1)k b . c Do x1 , x2 , . . . , xk cũng là nghiệm của P (x) nên an xni = −an−1 xin−1 + · · · + a1 xi + a0 , ∀i = 1, k. Nhân k đẳng thức này theo vế, ta thu được k Y n−1 k n |an | .|x1 x2 . . . xk | = (an−1 xi + · · · + a1 xi + a0 ) , i=1 hay  n Y k b k a . = (an−1 xn−1 + · · · + a1 xi + a0 ) i n c i=1 (1). Theo định lí Vieta, ta thấy tất cả các đa thức cơ bản của x1 , x2 , . . . , xk đều là các số hữu tỉ có mẫu chung là an . Vì thế tất cả các đa thức đối xứng với hệ số nguyên của các biến Q x1 , x2 , . . . , xk cũng là số hữu tỉ mà mẫu số là lũy thừa nào đó của an . Rõ ràng, ki=1 (an−1 xin−1 + · · · + a1 xi + a0 ) là một đa thức đối xứng với hệ số nguyên của các biến x1 , x2 , . . . , xk nên k Y M n−1 (a x + · · · + a x + a ) , n−1 i 1 i 0 = |an |m i=1 (2) . . trong đó M, m ∈ N. Hơn nữa, do a0 , a1 , . . . , an−1 .. p nên M .. p. Thay (2) vào (1) được  n k b m+k n M n a . n c = |an |m ⇐⇒ an .b = M|c| . 20 . .bn .. p. Vô lí ! Vậy điều giả ban đầu là sai hay P (x) là một đa Từ đây suy ra am+k n thức bất khả quy. Chú ý. Nhờ vào công cụ số phức ta có thể chứng minh được tiêu chuẩn Eisenstein mở rộng. ” Cho đa thức P (x) = an xn + an−1 xn−1 + · · ·+ a1 x+ a0 ∈ Z[x]. Nếu tồn tại số nguyên tố p và số nguyên 0 ≤ k ≤ n sao cho 3 điều kiện sau đây đồng thời thỏa mãn i) a0 , a1 , ..., ak−1 chia hết cho p; ii) ak không chia hết cho p; iii) a0 không chia hết cho p2 thì trong phân tích P (x) thành nhân tử có bậc lớn hơn hoặc bằng k.” Định lý 2 (Tiêu chuẩn Cohn). Cho p là một số nguyên tố và an an−1 . . . a1 a0 là biểu diễn của p trong hệ cơ số b (b ≥ 2). Khi đó P (x) = an xn + · · · + a1 x + a0 là một đa thức bất khả quy. Chứng minh. Để chứng minh tiêu chuẩn này ta cần 2 bổ đề sau n Bổ đề 1. Cho đa  thức f (x) = an x +· · ·+a1 x+a0 ∈ Z[x] thỏa mãn an ≥ 1, an−1 ≥ 0. Đặt M = max |ai |, i = 1, n − 2 . Khi đó nếu α là một nghiệm của f (x), thì hoặc √ 1 + 1 + 4M Re(α) ≤ 0 hoặc |α| < . 2 Chứng minh. √ Giả sử ngược lại rằng f (x) có một nghiệm α sao cho Re(α) > 0 và 1 + 1 + 4M . Vì Re(α) = Re(α) > 0 nên |α| ≥ 2   f (α) = an + an−1 + an−2 + · · · + a0 ≥ an + an−1 − |an−2 | + · · · + |a0 | αn α α2 αn α |α|2 |α|n    1 1 an−1  −M +···+ ≥ Re an + 2 α |α| |α|n ! 1 1 − |α|n−1 an−1 = an + .Re(α) − M. |α|2 |α|2 − |α| >1− M |α|2 − |α| − M = ≥ 0. |α|2 − |α| |α|2 − |α| Điều này vô lí, vì α là nghiệm của f (x).

- Xem thêm -

Tài liệu liên quan